Solving the Hula-hoop Problem: Can it be Done?

  • Thread starter wrobel
  • Start date
In summary: So the angular momentum vector in the S frame (relative to the inertial frame) is just the vector sum of the angular momentum vectors in the x, y, and z directions?
  • #1
wrobel
Science Advisor
Insights Author
1,104
960
I would like to propose an olympic (as it seems to me) level problem.

There is a vertical pillar of radius r and a thin hoop of mass m and radius R( R>r).
The hoop putted on the pillar rotates such that the center of the hoop describes a circle in horizontal plan and the velocity of the center remains constant in its absolute value. The is no slipping between the hoop and the pillar. The hoop does not slides down being undergone with the gravity.
Show that if R>4r/3 then, for some suitable values of the velocity and the friction coefficient between the hoop and the pillar, the described above motion is possible. And if R<4r/3 then independently on the friction and the velocity such a motion does not exist.

90831067a4e1.png


What do you think about the problem? I do not ask a solution :) I just expect teaching professional comments
 
Science news on Phys.org
  • #2
wrobel said:
I would like to propose an olympic (as it seems to me) level problem.

There is a vertical pillar of radius r and a thin hoop of mass m and radius R( R>r).
The hoop putted on the pillar rotates such that the center of the hoop describes a circle in horizontal plan and the velocity of the center remains constant in its absolute value. The is no slipping between the hoop and the pillar. The hoop does not slides down being undergone with the gravity.
Show that if R>4r/3 then, for some suitable values of the velocity and the friction coefficient between the hoop and the pillar, the described above motion is possible. And if R<4r/3 then independently on the friction and the velocity such a motion does not exist.

90831067a4e1.png


What do you think about the problem? I do not ask a solution :) I just expect teaching professional comments
Welcome to the PF.

Are you a teacher asking about the difficulty level of this problem? Or are you a student looking for help on a schoolwork problem? We are trying to figure out which sub-forum here at the PF your thread fits into the best. :smile:
 
  • #3
Are you a teacher asking about the difficulty level of this problem?

yes I am
 
Last edited by a moderator:
  • #4
Did you solve it?

It is certainly beyond the typical homework problem. After thinking about the problem for 10 minutes, I don't see why there should be a minimal radius. The coefficient of friction and the angular velocity would have to be large, but it should work out.

Edit: Wait, found a mistake.
Edit2: Fixed, didn't change the conclusion.
 
  • #5
I composed this problem for an olympiad in theoretical mechanics not for ordinary homework. To solve it one must write down the equations of the rigid body motion and look at them
 
  • #6
That's what I did.
Setting the cylinder radius to 1, and with ##\theta## as tilt of the hoop relative to the horizontal, I get ##\tan (\theta) \omega^2 (R \cos (\theta) - 1) = g## as condition for the angle. With sufficiently large ω this has a solution for all R>1. It will also satisfy the geometric boundary condition of ##R \cos (\theta) > 1##. The required coefficient of friction is ##\mu \geq \tan (\theta)##.
 
  • #7
That's what I did.
 

Attachments

  • hula-hoop-eng.pdf
    36 KB · Views: 563
  • #8
I guess [a,b] means the cross product?
I think that rotating coordinate system makes everything more complicated than necessary. The hoops angular velocity in the S frame should be constant (it is always in the z direction with constant magnitude), and if you drop the acceleration term there the constraint on R disappears.
 
  • #9
mfb said:
I guess [a,b] means the cross product?
yes

mfb said:
The hoops angular velocity in the S frame should be constant (it is always in the z direction with constant magnitude)
sure, this is exactly what was written. But the angular velocity vector rotates relative to the inertial frame. The equation of angular momentum is also written with respect to the inertial frame, this equation is just expanded in the basis $e_x,e_y,e_z$ .
 
  • #10
wrobel said:
But the angular velocity vector rotates relative to the inertial frame.
That's what the other ##\omega## does, right?
Hmm, confusing.
 

1. Can the hula-hoop problem be solved?

Yes, it is possible to solve the hula-hoop problem. However, it depends on various factors such as the size and weight of the hula-hoop, the strength and technique of the person using it, and environmental conditions.

2. What is the hula-hoop problem?

The hula-hoop problem refers to the challenge of keeping a hula-hoop spinning around the body for an extended period. It requires coordination, balance, and skill to maintain the hula-hoop's momentum and prevent it from falling to the ground.

3. What are some tips for solving the hula-hoop problem?

Some tips for solving the hula-hoop problem include starting with a larger and lighter hula-hoop, keeping the body relaxed and moving in a circular motion, and practicing with both the waist and hands to maintain the hula-hoop's momentum. It also helps to use a hula-hoop with grip tape or add grip tape to the hula-hoop for better control.

4. Can anyone solve the hula-hoop problem?

While there is no guarantee that everyone can solve the hula-hoop problem, anyone can improve their chances by practicing and using the right technique. Some people may have a natural knack for hula-hooping, while others may need more practice to master the skill.

5. What are the benefits of solving the hula-hoop problem?

Solving the hula-hoop problem can have various benefits, including improving coordination, balance, and core strength. It can also be a fun and enjoyable form of exercise and can help relieve stress and boost mood. Additionally, mastering the hula-hoop can give a sense of accomplishment and boost self-confidence.

Similar threads

  • STEM Educators and Teaching
2
Replies
36
Views
5K
  • Special and General Relativity
Replies
5
Views
1K
  • Introductory Physics Homework Help
Replies
1
Views
3K
  • Introductory Physics Homework Help
Replies
9
Views
2K
  • Advanced Physics Homework Help
Replies
2
Views
2K
  • Introductory Physics Homework Help
Replies
25
Views
4K
  • Introductory Physics Homework Help
Replies
6
Views
4K
Replies
15
Views
1K
  • Advanced Physics Homework Help
Replies
5
Views
2K
  • Introductory Physics Homework Help
Replies
4
Views
2K
Back
Top